[PDF] PRACTICE QUESTIONS TO HELP YOU MASTER THE PART I FRM EXAM





Previous PDF Next PDF



FOI question paper 2016

(1) Expected return of the portfolio. (ii) Minimum risk of the portfolio. (iii) Maximum risk of the portfolio. Solution: (b). (1) E(R) = 0.3 X 13 +0.70 X 16.



Consolidated questions and answers (Q&A) on the PRIIPs Key Consolidated questions and answers (Q&A) on the PRIIPs Key

2023年5月17日 ... portfolio may be obtained including ... that not all of the return history of the fund may be representative of the fund's current overall risk.



ESMA34-43-392 Q&As on UCITS Directive ESMA34-43-392 Q&As on UCITS Directive

8. ESMA will review these questions and answers on a regular basis to identify if in a certain portfolio credit risk would remain un-hedged. Answer 1a: Yes ...



Questions and Answers Questions and Answers

2017年4月4日 ... portfolio risk ratio to 3 which is consistent with your risk ... product's expected return in a “moderate” scenario as referred to in ...



Questions and Answers

2023年6月14日 AIFM to whom portfolio management or risk management activities have been delegated? ... expected annual return/IRR in normal market conditions?



Suggested Answer_Syl12_Jun2014_Paper_14

SUGGESTED ANSWERS TO QUESTIONS. JUNE 2014. Paper- 14: ADVANCED FINANCIAL Expected Return on the Portfolio in part (ii) is : RP = [3 × 0.20] + [( - )1 ...



Exam IFM Sample Questions and Solutions Finance and Investment

Assume the expected return of the portfolio is 0.12 the annual effective risk-free rate is 0.05



PORTFOLIO RISK AND RETURN PART I

In portfolio composition questions return and standard deviation are the key variables. Here you are told that both returns and standard deviations are 



Individual investors information use subjective expectations

https://www.diva-portal.org/smash/get/diva2:1305567/FULLTEXT02.pdf



Spotlight Quiz - Portfolio Theory and Risk Worked Solutions

In combining assets with different risk / return characteristics which of the following generates the advantage that the portfolio can provide over investments 



FOI question paper 2016

Portfolio Risk. ?(0.5)² (73.6) + (0.5)²(40) + 2(.5)(.5)(-52). = ?18.4+10-26. = ?2.4. = = 1.55 %. Hence risk will be reduced if the financial analyst 



Suggested Answer_Syl12_Jun2014_Paper_14

From Section A: Answer any two questions. From Section B: Answer any portfolio with a standard deviation of 24% what is the return of such portfolio?



Investment Analysis and Portfolio Management

return and risk in the same portfolio? And what could be the influence of this relationship to the investor's portfolio? The answers to these questions are 



Exam IFM Sample Questions and Solutions Finance and Investment

Calculate the standard deviation of the portfolio return. The other answer choices are more about alternative risk preferences (B) proxy error (C)



Questions and Answers

Jul 20 2022 Question 4: Efficient portfolio management techniques . ... Question 5: Calculation of counterparty risk for exchange-traded derivatives and.



Practice Problems for the Final Examination

Which of the following is a measure of the systematic risk of a stock? Portfolio A: expected return of 10% and standard deviation of 8%.



18 UBM 306 FINANCIAL MANAGEMENT Multiple Choice Questions

ANSWER: C. 11.Risk-return trade off implies_____________. A. Increasing the portfolio of the firm through increased production. B. Not taking any loans which 



Answers to Discussion Questions

What does modern portfolio theory (i.e. traditional finance) say about how an Next



TYBAF- SEM -6 SAPM MCQ (Bold = Correct Answer)

The efficient frontier is the set of ______ portfolios that offers the highest expected return for a defined level of risk on the lowest risk for a given 



Chapter 6 -- Interest Rates

Risk premium: additional return to compensate for additional risk Answer: yield on 1-year bond r1 = 3% + 2% = 5%; yield on 3-year bond



CHAPTER 11 RISK AND RETURN: THE CAPITAL ASSET PRICING MODEL

RISK AND RETURN: THE CAPITAL ASSET PRICING MODEL (CAPM) Answers to Concepts Review and Critical Thinking Questions Some of the risk in holding any asset is unique to the asset in question By investing in a variety of assets this unique portion of the total risk can be eliminated at little cost



Concept of Risk-Return in Portfolio Context (With Formulas)

risk and if some of the investments do not pan out the others will keep the value of the portfolio intact The two main features of a portfolio are its risk and expected return In 1952 Harry Markowitz first developed the ideas of portfolio theory based upon statistical reasoning



PRACTICE QUESTIONS TO HELP YOU MASTER THE PART I FRM EXAM

Answer: A Systematic risk is the risk that can’t be diversified away and the beta of our portfolio is: ? = (? PM* ? * ?M) / ?2 where ? PMis the correlation coeffi cient between the portfolio and the market ? is the risk of the portfolioand ? is the risk of the market



RISK AND RETURN - Cengage

This chapter explores the relationship between risk and return inherent in investing in securities especially stocks In what follows we’ll define risk and return precisely investi- gate the nature of their relationship and find that there are ways to limit exposure to in- vestment risk



CHAPTER 2 RISK AND RATES OF RETURN - umledu

Portfolio return Answer: a 8 An investor is forming a portfolio by investing $50000 in stock A that return on the market is equal to 6 percent and Treasury bonds have a yield of 4 percent What is the required rate of return on the investor’s portfolio? a 6 6 b 6 8 c 5 8 d 7 0 e 7 5 Portfolio return Answer: b 9



Searches related to portfolio risk and return questions and answers filetype:pdf

Answers 2-1 Stand-alone risk is the risk faced by an investor who holds just one asset versus the risk inherent in a diversified portfolio Stand-alone risk is measured by the standard deviation (SD) of expected returns or the coefficient of variation (CV) of returns = SD/expected return

What are the risk-return characteristics of a portfolio?

    Each portfolio has risk-return characteristics of its own. A portfolio comprising securities that yield a maximum return for given level of risk or minimum risk for given level of return is termed as ‘efficient portfolio’.

What is risk-return in portfolio context?

    Concept of Risk-Return in Portfolio Context (With Formulas) 1 i. Portfolio Return: The expected return of a portfolio represents weighted average of the expected returns on the securities comprising that portfolio with weights being the proportion of total funds ... 2 ii. ... 3 iii. ... 4 iv. ...

What are ask portfolio returns?

    ASK Portfolio returns are composite returns of all the Portfolios aligned to the investment approach as on Mar 31, 2023. Returns for individual client may differ depending on time of entry in the Portfolio. Past performance may or may not be sustained in future and should not be used as basis for comparison with other investments.

What are the assumptions of the portfolio theory?

    Another assumption of the portfolio theory is that the returns of assets are normally distributed which means that the mean (expected value) and variance analysis is the foundation of the portfolio. i. Portfolio Return:

Wiley © 2016

PRACTICE QUESTIONS

TO HELP YOU MASTER

THE PART I FRM

EXAM

FRM® Exam Review

Top questions you must master to pass the Part I FRM® Exam Preparing for the Part I exam is tough, but you can make life easier with an effective study plan. If you have yet to get a plan, Wiley's adaptive Digital Exam Planner in our Silver and Self-Study FRM® review courses will help you create a personalized plan down to the day, provide a dashboard to keep on track and track your progress every step of the way. But first, here are some questions to test your knowledge of typical, fundamental topics that are likely to appear on the actual exam. 1. The minimum variance frontier most likely consists of: A.

Individual assets only.

B.

Portfolios only.

C.

Individual assets and portfolios.

D.

Only risk-free assets.

Answer: B

Assets with low correlations can be combined into

portfolios that have a lower risk than any of the individual assets in the portfolio. The minimum variance frontier consists of portfolios that minimize the level of risk for each level of expected return. 2. Compute her portfolio's standard deviation, if the correlation between the two assets equals 0.7. A. 8.05% B. 9.86% C. 7.06% D.

12.68%

Answer: D

[(0.3 2

× 0.12

2 ) + (0.7 2

× 0.0

2 ) + 2 (0.3) (0.7) (0.12) (0.1414) (0.7)] 0.5 = 12.68% 3. Use the following information to answer the next four questions:

The following information is available regarding the portfolio performance of three investment managers:

ManagerReturnStandard

DeviationBeta

A19%27%0.7

B14%22%1.2

C16%19%0.9

Market (M)11%24%

Risk-free rate5%

I. Manager B's expected return is

closest to: A. 9.20% B. 8.34% C.

10.40%

D.

12.20%

Answer: D

Expected return =

R f R R m R f Expected return = 0.05 + 1.2 (0.11 - 0.05) = 12.20% II.

Manager A's Sharpe ratio is closest to:

A. 0.51 B. 0.40 C. 0.20 D. 0.68

Answer: A

Sharpe ratio = (

R A R f R A = (0.19 - 0.05) / 0.27 = 0.5185 III.

Manager C's Treynor ratio is closest to:

A. 0.20 B. 0.25 C. 0.57 D. 0.12

Answer: D

Treynor ratio = (

R C R f R C = (0.16 - 0.05) / 0.9 = 0.1222 IV.

Manager C's Jensen's alpha is closest to:

A. 5.60% B.

10.40%

C. 8.5% D. 9.0%

Answer: A

Manager C's expected return =

R f R R m R f ) = 0.05 + 0.9 (0.11 - 0.05) = 10.4%

FRM® Exam Review

4. Darren Peters, FRM, has gathered information on all the monthly returns of actively managed portfolios and passive indices. He is using multifactor models, of which he has examined many. Darren determines the optimal models. The following results were derived for the historical data:

RTN = -.0025 + .15MKT - .08EPSS - .07LCSC

Which of the following is not a reason to support the case for active portfolio management? A.

Failure of the CAPM beta to explain returns

B.

Excess volatility in market prices

C.

The existence of market anomalies

Answer: D

All are valid reasons to support the case for active portfolio allocate their money between the risk-free asset and the 5. A.

Treynor ratio

B.

Sharpe ratio

C.

Jensen's alpha

D.

Sortino ratio

Answer: A

and the beta of our portfolio is: P PM P M

2 where

PM cient between the portfolio and the market, p is the risk of the portfolio, and M is the risk of the market. In either case, beta explains the volatility of the portfolio compared to the volatility of the market, which captures only systematic risk.The Sharpe ratio is standardized by sigma, not beta, so the Treynor ratio is the correct ratio to use in this case. The Treynor formula is T E R R f] / systematic risk - the beta - which is what the question asks. 6. A.

4.13% to 4.37%

B.

4.22% to 4.44%

C.

4.14% to 4.36%

Answer: A

Since the population variance is not known, but the population is assumed to be normally distributed, and the sample size is small we must use the t -distribution.

Standard error = 0.3 / (20)

0.5 = 0.06708

The relevant

t -score with 19 degrees of freedom is 1.7291. = 4.25% ± (1.7291 × 0.067%) = 4.13% to 4.37% 7. A.

Reject the null hypothesis, and conclude that the

B. C.

Answer: C

H 0 : μ = 0; H a Since the population (index comprising of all shares traded in the country) standard deviation is known, use the z -test. critical z -values for a two-tailed test are ± 1.96. 0.5 ]} = 10.607

FRM® Exam Review

Since the test stat (10.607) is greater than the upper critical value (+1.96), the null hypothesis is rejected. Alexis would conclude that the mean daily return is statistically questions:

An analyst regresses the bid/ask spread (dependent variable) for a sample of 1,900 stocks against the natural log of trading volume (independent variable). The results of the regression are provided below:

ANOVASS

Regression18.395

Residual47.428

Errort-Statistic

Intercept0.629410.02663523.63094

A.

0.2795

B.

0.3879

C.

0.7205

Answer: A

47.428) = 0.2795

A.

0.6228

C.

0.5286

Answer: B

0.5 0.5 = 0.5286 r III.

The standard error is closest to:

A.

0.0984

quotesdbs_dbs10.pdfusesText_16
[PDF] portfolio standard deviation calculator

[PDF] portfolio standard deviation excel

[PDF] portfolio standard deviation formula

[PDF] portion sizes for toddlers 1 3 years

[PDF] portland area bike routes

[PDF] portland bike map app

[PDF] portland bike rides 2018

[PDF] portland maine police arrests records

[PDF] portland maine police beat

[PDF] portland maine police department non emergency number

[PDF] portland maine police staff

[PDF] portland police academy

[PDF] portland police activity log

[PDF] portland police department maine arrest log

[PDF] portland police recruitment